Download as docx, pdf, or txt
Download as docx, pdf, or txt
You are on page 1of 39

AAO Uveitis 01 Short

Total Questions: 6 (6 Points)


Correctly Answered Questions: 0 (0 Points)
Percentage Score 0%

Questions

Question: A patient presents with meningismus, vitiligo, and bilateral exudative retinal detachments.
What diagnosis is most consistent with these findings? (1 Points)
Answer:

A. Fuchs heterochromic iridocyclitis (0 Points)


B. Human leukocyte antigen (HLA)-B27 uveitis (0 Points)
C. Vogt-Koyanagi-Harada syndrome (1 Points)

Answer Feedback:
Vogt-Koyanagi-Harada syndrome is a multi-system disorder featuring ocular inflammation
(often with exudative detachments), CNS findings, such as meningismus, and auditory or
vestibular findings. Later in the diesease course, skin is involved including vitiligo, poliosis,
and vitiligo. It is always bilateral, wherase herpes simplex, Fuchs heterochromic cyclitis, and
HLA-B27 uveitis more typically present unilaterallyl.

D. Herpes simplex uveitis (0 Points)

Your answer: (0 Points)

Question: A 27-year-old man presents with a history of floaters and blurred vision OD. The patient
immigrated to the United States from India within the last year, is currently in good health, and denies
any history of drug use. Two years ago, while in India, he had a positive tuberculin skin test, and
received 9 months of prophylactic isoniazid, rifampin, and ethambutal therapy. Anterior segment
examination is normal. The right fundus is shown in the figure. The left fundus showed findings similar
to those of the right. Vitreous hemorrhage is noted in the inferior vitreous OD. No pars plana
snowbanks are noted in either eye. Which of the following would be least helpful in the workup of this
patient? (1 Points)
Answer:

A. HLA-A29 (1 Points)

Answer Feedback:
This patient has bilateral retinal neovascularization, which may arise from a number of
conditions, including diabetes mellitus, branch retinal vein occlusion (BRVO), sickle-cell
retinopathy, Eales' disease, or from inflammatory conditions such as sarcoidosis, tuberculosis
(TB), Behcet's disease, peripheral uveitis, or systemic lupus erythematosus (SLE). Workup
should be directed toward these diagnoses and should include a serum glucose and/or
glucose tolerance test, fluorescein angiography (for BRVO), hemoglobin electrophoresis (for
sickle-cell retinopathy), chest xray (for TB, sarcoid), ACE titer (for sarcoid), ANA titer (for
SLE), HLA-B5 (for Behcet's disease), and a tuberculin skin test with anergy panel (for TB and
sarcoid). HLA-A29 would not need to be tested in this patient, because it is associated with
birdshot retinochoroidopathy, which produces a retinochoroiditis, but not retinal
neovascularization.

B. Serum glucose (0 Points)


C. Angiotensin-converting enzyme (ACE) and antinuclear antibody (ANA) titers (0 Points)
D. Hemoglobin electrophoresis (0 Points)

Your answer: (0 Points)

Question: A 30-year-old woman with a history of sarcoidosis presents with the retinal finding shown in
the figure. Which of the following statements is least accurate regarding this patient? (1 Points)
Answer:

A. Retinal neovascularization, which may accompany this condition, typically does not respond
to systemic corticosteroids or scatter photocoagulation. (1 Points)

Answer Feedback:
Sarcoidosis is a multisystem, granulomatous disease that most commonly affects the lungs,
skin, lymph nodes, and eyes. Pulmonary involvement, which may involve the lung
parenchyma or hilar lymph nodes, may affect 90% of patients with sarcoid. Therefore, a chest
x-ray may be helpful in detecting patients with the disease. Other laboratory tests that may be
helpful in making the diagnosis include serum angiotensin-converting enzyme levels, which
may be elevated in 60%-90% of patients; skin testing, which may demonstrate anergy in 50%
of patients; and whole-body gallium scanning, which may demonstrate uptake in areas of
active inflammation. Ocular involvement may be seen in 33%-50% of patients with sarcoid.
Patients with ocular involvement most often present with a granulomatous, anterior uveitis,
but may have a myriad of manifestations, including cataract, glaucoma, dry eye, conjunctival
nodules, lacrimal gland enlargement, vitritis, retinal periphlebitis, chorioretinitis, cystoid
macular edema, peripheral uveitis, optic disc edema, and retinal neovascularization. Patients
who have posterior uveitis are believed to have a higher incidence of central nervous system
involvement (35%) than patients without (5%). Topical corticosteroids are usually effective in
treating the anterior uveitis from sarcoidosis, but posterior disease usually requires periocular
or systemic corticosteroids. Retinal neovascularization secondary to sarcoidosis has also
been shown to respond to systemic corticosteroids. Retinal neovascularization secondary to
retinal ischemia may respond to scatter laser photocoagulation.

B. This patient is likely to demonstrate hilar lymphadenopathy and/or parenchymal infiltrates on


chest x-ray. (0 Points)
C. This patient has an increased risk of central nervous system involvement. (0 Points)
D. This patient may not demonstrate anergy on tuberculin skin testing. (0 Points)

Your answer: (0 Points)

Question: The ultrasonogram shown in the figure was obtained from an 18-year-old man who
presented with complaints of ocular pain, redness, mild proptosis, and blurred vision OD of 2 weeks
duration. Which of the following would be least helpful in the diagnostic workup of this patient? (1
Points)
Answer:

A. Computerized tomography of the orbits (0 Points)


B. HLA typing for Behçet's disease (1 Points)

Answer Feedback:
Because this patient presents with proptosis, redness, and pain, orbital causes, such as
diffuse idiopathic orbital inflammation (pseudotumor) and thyroid ophthalmopathy, must still
be carefully considered despite the ultrasonographic findings of a T sign. Therefore,
computed tomography may be helpful to rule out orbital disease. In fact, idiopathic orbital
inflammation may be very difficult to discern from posterior scleritis in patients who present
with proptosis, chemosis, and limitation of ocular movements. Ultrasonography and computed
tomography may show scleral thickening and retrobulbar edema, in addition to extraocular
muscle enlargement in either condition. Systemic disease has been found in 45% of patients
with posterior scleritis, including rheumatoid arthritis, systemic lupus erythematosus (SLE),
psoriatic arthritis, giant cell arteritis, and gout. Therefore, laboratory tests such as ANA (for
SLE and rheumatoid arthritis), rheumatoid factor (for rheumatoid arthritis), erythrocyte
sedimentation rate (for giant cell arteritis), uric acid (for gout), and limb xrays (for rheumatoid
arthritis, psoriatic arthritis, and gout) are helpful. Behçet's disease has not been associated
with posterior scleritis and has only rarely been associated with anterior scleritis; therefore,
HLA typing for Behçet's disease would be of little value.

C. Antinuclear antibodies (ANA) (0 Points)


D. Rheumatoid factor (0 Points)

Your answer: (0 Points)

Question: A 73-year-old man complains of increasing pain and decreasing vision OD over the last 3
days. Two weeks ago he underwent an uncomplicated phacoemulsification, and placement of a
foldable posterior chamber silicone intraocular lens (IOL) in the same eye. On examination, his visual
acuity is HM 00 and 20/30 OS. Intraocular pressures are normal. Slitlamp examination OD reveals
moderate conjunctival injection, a Seidel negative cataract wound, a 1.5-mm hypopyon in the anterior
chamber, and a fibrin plaque over the anterior surface of the IOL. A dull red reflex is present, and
fundus details are not visible. Ultrasonography shows diffuse vitreous debris, but no retinal or choroidal
detachment. The patient's findings are consistent with postoperative endophthalmitis. The patient
experienced decreasing inflammation and noted improved visual acuity for the first 2 days following
treatment. On the third post treatment day, however, visual acuity decreased and a new vitreous
hemorrhage was noted. Anterior segment examination demonstrated minimal corneal epithelial edema,
mild cell and flare in the anterior chamber, a residual fibrin plaque on the anterior surface of the IOL,
which is centered, and an open posterior capsule. Ultrasonography revealed a retinal detachment. Pars
plana vitrectomy, air-fluid exchange, and laser retinopexy are planned to repair the retinal detachment,
because the vitreous hemorrhage prevents an adequate view for scleral buckling. Which of the
following conditions in this patient may present the most difficulty for completing the planned
treatment? (1 Points)
Answer:

A. Residual fibrin plague on the anterior surface of the IOL (0 Points)


B. Presence of a silicone IOL (1 Points)

Answer Feedback:
There is a risk of retinal detachment following pars plana vitrectomy or vitreous tap. This risk
is increased in patients with endophthalmitis, because the infection and resultant
inflammation may induce areas of retinal weakening, vitreous organization, and increased
vitreoretinal adhesion. The vitreous hemorrhage in this patient is most likely the result of a
retinal break development. Visualization of the retina in this patient may be made difficult by
epithelial corneal edema, residual fibrin on the IOL, residual vitreous debris, and vitreous
hemorrhage. However, the corneal epithelium may be debrided, the fibrin plaque stripped
from the IOL, and vitrectomy will remove the vitreous opacities such that none of these should
pose a significant obstacle in repairing the retinal detachment as planned. The use of prior
intravitreal antibiotics should not directly affect the performance of the vitrectomy. The
patient's clinical response to the initial treatment for endophthalmitis suggests the infection
was adequately treated and may not require further intravitreal antibiotics, which would not be
possible with the use of intravitreal air. The presence of a silicone intraocular lens, however,
may pose a significant problem with visualization during vitrectomy once an air-fluid exchange
is performed. Frequent condensation like fluid deposits have been noted to occur on the
posterior surface of foldable silicone IOLs with open posterior capsules. This condensation
may severely impair the view of the retina during the air-fluid exchange and may be difficult to
clear during surgery. Obviously, endophthalmitis and postoperative retinal detachment cannot
be anticipated, but the use of foldable silicone IOLs should be carefully considered in patients
who may require later retinal surgery.

C. Previous intravitreal antibiotics (0 Points)


D. Vitreous hemorrhage (0 Points)

Your answer: (0 Points)


Question: A 73-year-old man complains of increasing pain and decreasing vision OD over the last 3
days. Two weeks ago he underwent an uncomplicated phacoemulsification, and placement of a
foldable posterior chamber silicone intraocular lens (IOL) in the same eye. On examination, his visual
acuity is HM 00 and 20/30 OS. Intraocular pressures are normal. Slitlamp examination OD reveals
moderate conjunctival injection, a Seidel negative cataract wound, a 1.5-mm hypopyon in the anterior
chamber, and a fibrin plaque over the anterior surface of the IOL. A dull red reflex is present, and
fundus details are not visible. Ultrasonography shows diffuse vitreous debris, but no retinal or choroidal
detachment. The patient's findings are consistent with postoperative endophthalmitis. Based on the
Endophthalmitis Vitrectomy Study (EVS) findings, what would be the recommended initial course of
treatment for this patient? (1 Points)
Answer:

A. Intravenous antibiotics, vitreous tap, and injection of intravitreal antibiotics (0 Points)


B. Immediate pars plana vitrectomy and injection of intravitreal antibiotics (0 Points)
C. Vitreous tap and injection of intravitreal antibiotics (1 Points)

Answer Feedback:
The EVS was a prospective, randomized clinical trial that examined the role of intravenous
antibiotics and immediate pars plana vitrectomy in the treatment of postoperative
endophthalmitis occurring within 6 weeks of cataract extraction or placement of a secondary
intraocular lens. All patients received intravitreal antibiotics. Patients were randomized into
four groups based on whether or not they received intravenous antibiotics, or underwent
immediate pars plana vitrectomy vs. intravitreal tap (PPV vs TAP). The results demonstrated
no difference in final visual acuity or media clarity with the use of systemic antibiotics.
Patients with an initial visual acuity of HM or better showed no difference in final visual acuity
whether or not immediate pars plana vitrectomy was performed. However, patients with an
initial visual acuity of light perception (LP) were found to benefit from immediate vitrectomy.
Patients with LP vision who received immediate vitrectomy were found to have a three-fold
increase in achieving 20/40 or better (33% PPV vs 11% TAP), a two-fold increase in
achieving 20/100 or better (56% PPV vs 30% TAP), and a 50% decrease in the frequency of
severe visual loss, defined as <5/200 (20% PPV vs 47% TAP). Therefore, based on the EVS
findings, this patient does not require intravenous antibiotics or immediate vitrectomy,
because his presenting visual acuity is HM.

D. Intravenous antibiotics, immediate pars plana vitrectomy, and injection of intravitreal


antibiotics (0 Points)

Your answer: (0 Points)

Print

AAO Uveitis 02 Short

Total Questions: 8 (8 Points)


Correctly Answered Questions: 0 (0 Points)
Percentage Score 0%

Questions
Question: Which of the following statements does not accurately describe sympathetic ophthalmia?
(1 Points)
Answer:

A. Phacoanaphylaxis has been noted in up to 25% of patients with sympathetic ophthalmia. (0


Points)
B. High-dose systemic corticosteroids are the initial treatment of choice in sympathetic
ophthalmia. (0 Points)
C. Sympathetic ophthalmia may be differentiated from Vogt-Koyanagi-Harada syndrome by the
absence of vitiligo and alopecia in the former. (1 Points)

Answer Feedback:
Sympathetic ophthalmia is a rare, bilateral, granulomatous uveitis that may occur following
ocular trauma or surgery. The onset of inflammation rarely occurs within 2 weeks of injury and
may not occur until decades after trauma. The majority (90%) of cases, however, begin within
1 year. Symptoms in sympathetic ophthalmia typically include pain, photophobia, blurred
vision, and decreased accommodation. Examination may reveal keratitic precipitates on the
cornea, anterior uveitis, vitritis, retinal edema, papillitis, and the yellow-white subretinal
pigment epithelium (RPE) Dalen-Fuchs nodules. Fluorescein angiography typically shows
multiple pinpoint areas of hyperfluorescence. Pathologic examination of eyes with
sympathetic ophthalmia has shown a frequent association of phacoanaphylaxis (up to 25%).
Sympathetic ophthalmia may be difficult to distinguish clinically from Vogt-Koyanagi-Harada
(VKH) syndrome, because they share not only similar ocular findings, but also systemic find-
ings such as vitiligo and alopecia. Vitiligo and alopecia, which are typical in VKH, may also be
seen in sympathetic ophthalmia, although less commonly. Debate still exists as to the benefit
of enucleating the exciting eye once sympathetic ophthalmia has begun. However, high-dose
systemic corticosteroids (100-200 mg) have been shown to be effective in the initial treatment
of sympathetic ophthalmia. In patients unable to tolerate systemic corticosteroids,
immunosuppressive agents such as methotrexate, azathioprine, or cyclosporin may be used.

D. Sympathetic ophthalmia rarely occurs within 2 weeks of injury. (0 Points)

Your answer: (0 Points)

Question: A 28-year-old man with acquired immunodeficiency syndrome (AIDS) presents with the
right fundus shown in the figure. Which of the following statements regarding the patient's condition is
most accurate? (1 Points)
Answer:

A. If the patient is without symptoms, and no retinal hemorrhage is associated with these
lesions, they do not represent CMV retinitis. (0 Points)
B. The lesions may represent either cotton-wool spots or early CMV retinitis. Close observation
of the patient for change in these lesions, with documentation of the fundus by photographs,
is indicated. (1 Points)

Answer Feedback:
These white retinal lesions may represent either cotton-wool spots, a manifestation of HIV-
related noninfectious retinal vasculopathy, or the earliest manifestation of CMV retinitis. Both
the retinal vasculopathy associated with HIV infections and CMV retinitis are typically late
manifestations of AIDS. CMV retinitis, in particular, is more common when CD4 lymphocyte
counts are reduced to 50cells/cubic mm or less. CMV retinitis often may present without
symptoms, and although the retinal lesions are characteristically areas of hemorrhagic retinal
necrosis, atypical presentations of CMV retinitis may occur. Therefore, the safest course in
this case is to document the fundus appearance with photographs, and observe closely for
any enlargement or change in these lesions, which would indicate CMV retinitis. Close
observation of the patient for change in these lesions, with documentation of the fundus by
photographs, is indicated.

C. The lesions most likely are cotton-wool spots, which are a manifestation of human
immunodeficiency virus (HIV) related noninfectious retinal vasculopathy, and do not require
close followup. (0 Points)
D. The lesions most likely represent early CMV retinitis if the patient's CD4 lymphocyte count is
less than 50 cells per cubic mm. (0 Points)

Your answer: (0 Points)

Question: Which of the following statements is not accurate about Propionibacterium acnes
endophthalmitis? (1 Points)
Answer:

A. Inflammation associated with Propionibacterium endophthalmibs typically increases with


topical corticosteroid use. (1 Points)

Answer Feedback:
Propionibacterium acnes is a Gram-positive, anaerobic, pleomorphic bacillus that is
commonly present in the normal conjunctival flora. It has been recognized as a cause of
chronic, recurrent, often granulomatous inflammation following cataract extraction. The
inflammation typically associated with Propionibacterium acnes may not develop for weeks to
months after uncomplicated cataract surgery. The organism may be sequestered in the
capsular bag and a characteristic capsular plaque may be visible. Flare-up of intraocular
inflammation has been reported following Nd:YAG laser capsulotomy, which may expose or
release more organisms. Inflammation from Propionibacterium acnes typically shows a
transient response to corticosteroids, but intraocular administration of antibiotic with or without
vitrectomy, capsulectomy, and/or IOL removal or exchange are the preferred treatments for
Propionibacterium acnes endophthalmitis.

B. Nd:YAG laser capsulotomy may cause a flare-up of the intraocular inflammation in


Propionibacterium endophthalmitis. (0 Points)
C. Propionibacterium acnes is frequently found in the normal flora of the conjunctiva. (0 Points)
D. Propionibacterium may cause a recurrent granulomatous iridocyclitis that may not develop
until months after cataract surgery. (0 Points)

Your answer: (0 Points)

Question: A 26-year-old myopic man presents with a 5-day history of photopsias, small scotomas,
and blurred vision in both eyes. He is recovering from a recent flu-like illness. Examination reveals
best-corrected visual acuity of 20/50 OD and 20/40 OS. Slit-lamp examination shows mild flare and cell
in both anterior chambers, and mild vitreous cell in both eyes. The fundus findings are similar in both
eyes; the right fundus is shown in the figure. Which of the following diagnoses is most likely in this
patient? (1 Points)
Answer:

A. Acute posterior multifocal placoid pigment epitheliopathy (0 Points)


B. Presumed ocular histoplasmosis syndrome (0 Points)
C. Multifocal choroiditis (1 Points)

Answer Feedback:
The clinical picture in this patient represents an inflammatory process of the choroid and
retina. Inflammatory retinal and choroidal diseases are classified based on ophthalmoscopic
findings and clinical course of the disease, with overlap between many of the diagnoses. Of
the choices given, multifocal choroiditis best fits the clinical history and appearance in this
patient. The patient's young age is consistent with any of the listed diagnoses except birdshot
retinochoroidopathy, which is more common in patients between ages 40 and 60. Bilateral
ocular involvement is seen in all of the diagnoses. A preceding viral illness is a frequent
history given by patients with either multifocal choroiditis or acute posterior multifocal placoid
pigment epitheliopathy. Vitritis is a finding in multifocal choroiditis, birdshot
retinochoroidopathy, and acute posterior multifocal placoid pigment epitheliopathy. Acute
posterior multifocal placoid pigment epitheliopathy causes multiple yellow-white, flat, round or
irregular lesions at the level of the pigment epithelium and choroid, typically larger than the
lesions present in this patient. Presumed ocular histoplasmosis syndrome commonly
produces peripapillary scarring (not present in this patient) in addition to typical punched-out,
peripheral chorioretinal scars and lack of vitreous cells.

D. Birdshot retinochoroidopathy (0 Points)

Your answer: (0 Points)

Question: A 30-year-old woman complains of blurred central vision in the right eye for the past 3
days. Multiple chorioretinal lesions are noted in both eyes. Which of the following findings is most
consistent with a diagnosis of acute posterior multifocal placoid pigment epitheliopathy (APMPPE)? (1
Points)
Answer:

A. Subretinal hemorrhage in the macula and peripheral scotomas (0 Points)


B. Clear vitreous and subtle disc edema (0 Points)
C. Vitreous cells and satellite lesions (0 Points)
D. Symptoms in the other eye within 2 days (1 Points)

Answer Feedback:
Possible diagnoses, besides APMPPE, are presumed ocular histoplasmosis syndrome
(POHS), recurrent multifocal choroiditis (pseudo POHS), and toxoplasmosis. While all of
these conditions have quite different clinical findings and presentations, variants of them can
sometimes be confusing. The typical POHS case has peripapillary atrophy of the retinal
pigment epithelium and peripheral histo spots; and, in the macula, choroidal
neovascularization with subretinal fluid and hemorrhage can occur. Bilaterality is common,
but usually years elapse between macular involvement of the first eye and that of the other
eye. Vitreous cells are not seen in typical POHS. Recurrent multifocal choroiditis mimics
POHS. It is distinguished clinically from POHS by mild panuveitis and frequent recurrences,
with the development of new fundus lesions. Unilateral cases may become bilateral.
Toxoplasmosis produces focal areas of chorioretinal inflammation, often adjacent to previous
areas of involvement (satellite lesions). Heavy vitreous cellular infiltration is the rule, and it is
unusual to have simultaneous bilateral involvement. APMPPE usually occurs following a viral
illness and is usually bilateral, with rapid involvement of both eyes once symptoms begin.
Spontaneous recovery of vision is usual.

Your answer: (0 Points)

Question: Choroidal hemangiomas are not often associated with which of the following findings? (1
Points)
Answer:

A. Subretinal serous or lipid exudation (0 Points)


B. A subretinal neovascular membrane (1 Points)

Answer Feedback:
Choroidal hemangiomas are benign vascular tumors that may arise as isolated lesions
without other vascular malformations, as diffuse thickening of the choroid in association with
Sturge-Weber syndrome, or a nonfamilial condition characterized by ipsilateral angiomatous
lesions involving the face, brain, and uveal tract. In contrast to melanomas, choroidal
hemangiomas exhibit high internal reflectivity on A-scan echography. During fluorescein
angiography, they fill during the prearteriole and arteriole phases and leak during the late
phases. Serous or lipid exudation accounts for loss of vision in many patients. Subretinal
neovascular membranes rarely, if ever, develop from choroidal hemangiomas.

C. Leakage of dye in the late phases of fluorescein angiography (0 Points)


D. High internal reflectivity on A-scan echography (0 Points)
Your answer: (0 Points)

Question: A-26 year-old Caucasian woman with bilateral intermediate uveitis of two years duration
complains of shooting pains down her leg after a hot bath. What is the most likely diagnosis? (1
Points)
Answer:

A. Behcets disease (0 Points)


B. toxoplasmosis (0 Points)
C. B-cell lymphoma (0 Points)
D. multiple sclerosis (1 Points)

Answer Feedback:
This symptom is Lehrmitte’s sign, which is found in patients with multiple sclerosis. The two
most common underlying entities associated with intermediate uveitis are multiple sclerosis
and sarcoidosis. The other entities can be considered, but are unlikely in this patient.

Your answer: (0 Points)

Question: What is the most common cause of decreased visual acuity from the inflammation in
idiopathic intermediate uveitis? (1 Points)
Answer:

A. Vitreous hemorrhage (0 Points)


B. Retinal detachment (0 Points)
C. Cystoid macular edema (1 Points)

Answer Feedback:
Multiple studies confirm cystoid macular edema is the most common cause of decreased
vision in patients with intermediate uveitis. Glaucoma is often due to corticosteroid treatment,
though a small number of patients can have sufficient anterior chamber inflammation to form
enough synechiae to result in a uveitic glaucoma by that mechanism. Vitreous hemorrhage
and retinal detachment can occur, presumably secondary to vitreous traction from the virtitis
or from organized vitreous secondary to previous inflammation.

D. Glaucoma (0 Points)

Your answer: (0 Points)

Print

AAO Uveitis 03 Short

Total Questions: 9 (9 Points)


Correctly Answered Questions: 0 (0 Points)
Percentage Score 0%
Questions

Question: What does the differential diagnosis of idiopathic pars planitis include? (1 Points)
Answer:

A. Vogt-Koyanagi-Harada syndrome (0 Points)


B. Rheumatoid arthritis (0 Points)
C. Ankylosing spondylitis (0 Points)
D. Lymphoma (1 Points)

Answer Feedback:
Pars planitis is a form of intermediate uveitis featuring vitritis, pars plana snowbanks, and
frequently cystoid macular edema. It is bilateral in 90% of cases. Intraocular lymphoma can
also present with bilateral vitritis, with or without a yellowish subretinal infiltrate. Ankylosing
spondylitis is associated with a recurrent acute unilateral anterior uveitis in most cases.
Rheumatoid arthritis is more commonly associated with scleritis and peripheral ulcerative
keratitis. Vogt-Koyanagi-Harada disease features bilateral granulomatous panuveitis,
frequently with exudative retinal detachment, meningismus and tinitus, and later in disease
course, integument findings.

Your answer: (0 Points)

Question: An 84-year-old woman with a history of bilateral pseudophakia presents with a 7-day
history of sudden, decreased vision in her right eye without associated pain, redness, or photophobia.
Examination reveals best corrected visual acuity of counting fingers at 2 feet OD and 20/25 OS.
Examination of the right eye shows an afferent pupillary defect, inferior keratic precipitates, and mild
flare and cell. The vitreous in the right eye contains clumps of white, cellular debris. Posterior chamber
IOLs are in position with central posterior capsular openings OU. The left eye is normal. The patient
was diagnosed 4 years ago with tuberculosis, and was treated medically for it for 1 year. Which one of
the following is least likely to be helpful in establishing a diagnosis in this patient? (1 Points)
Answer:

A. Vitreous biopsy for cytology and microbiology (0 Points)


B. Magnetic resonance imaging (MRI) of the head and orbits (0 Points)
C. Fluorescein angiography (1 Points)

Answer Feedback:
This patient presents with panuveitis, including vitritis, retinal vasculitis, and retinitis, with
involvement of the optic nerve head. Given the patient's age, clinical presentation, ocular
findings, and previous history of tuberculosis, an infectious or neoplastic cause is most likely.
Tuberculosis would be high in the differential diagnosis. Other organisms, particularly fungal,
should also be strongly considered. The patient is not known to be immunosuppressed, but a
thorough evaluation, including complete blood count with differential and HIV testing, should
be performed. Evidence of extraocular malignancy or active tuberculosis should also be
evaluated. MRI may provide information about involvement of the optic nerve or central
nervous system. Vitrectomy for vitreous biopsy should be performed in this patient, and the
specimen sent for both microbiologic and cytologic examination. Although there is clinical
evidence of retinal vascular inflammation, fluorescein angiography is unlikely to provide any
useful information in establishing a diagnosis.

D. Referral to an internist for evaluation of possible systemic malignancy, immunosuppression,


or reactivation of tuberculosis (0 Points)

Your answer: (0 Points)

Question: Which of the following diseases is not associated with HLA-B27 positivity? (1 Points)
Answer:
A. Rheumatoid arthritis (1 Points)

Answer Feedback:
Ankylosing spondylitis, reactive arthritis, psoriatic arthritis, inflammatory bowel disease, and
undifferentiated spondyloarthropathy are all HLA-B27 associated diseases that may all be
associated with anterior uveitis. Rheumatoid arthritis does not have a strong Class I HLA
association, and is more typically associated with scleritis or keratitis than anterior uveitis.

B. Reactive arthritis (0 Points)


C. Psoriatic arthritis (0 Points)
D. Ankylosing spondylitis (0 Points)

Your answer: (0 Points)

Question: Which form of juvenile idiopathic arthritis is most likely to be associated with anterior
uveitis? (1 Points)
Answer:

A. Polyarticular (0 Points)
B. Still's disease (0 Points)
C. Pauciarticular (1 Points)

Answer Feedback:
Between 25% and 40% of ANA+, pauciarticular onset patients with juvenile arthritis will
develop uveitis. The uveitis associated with this disorder tends to be asymptomatic, but very
persistent, and is associated with very high rates of complication.

D. ANA-; RF+ (0 Points)

Your answer: (0 Points)

Question: Which of the following parasites is associated with diffuse unilateral subacute
neuroretinitis? (1 Points)
Answer:

A. Baylisascaris procyonis (1 Points)

Answer Feedback:
DUSN is an uncommon condition usually presenting with visual changes, photopsias,
floaters, and progressing to field loss with a 'unilateral wipeout' appearance to the fundus,
similar to retinitis pigmentosum. The roundworm Baylisascaris procyonis, which is carried by
raccoons, has been implicated in this condition.

B. Taenia solium (0 Points)


C. Toxocara catis (0 Points)
D. Toxoplasma gondii (0 Points)

Your answer: (0 Points)

Question: Sterility is a side effect associated with which of the following immunomodulatory drugs?
(1 Points)
Answer:
A. Cyclosporine A (0 Points)
B. Mycophenolate mofetil (0 Points)
C. Chlorambucil (1 Points)

Answer Feedback:
The alkylating drugs chlorambucil and cyclophosphamide are associated with a dose-
dependent risk of sterility in both men and women. Sperm and ova banking should be
discussed with patients requiring these medications.

D. Azathioprine (0 Points)

Your answer: (0 Points)

Question: Which of the following is less likely to require immunomodulatory treatment in addition to
steroids? (1 Points)
Answer:

A. Wegener's granulomatosis (0 Points)


B. Mucous membrane pemphigoid (0 Points)
C. Sarcoidosis (1 Points)

Answer Feedback:
Certain uveitic conditions including Wegener's granulomatosis, mucous membrane
pemphigoid, Behcet's disease, and foveal-threatening serpiginous choroiditis need to be
treated from onset with immunomodulatory drugs. Sarcoidosis frequently responds to
corticosteroid therapy alone.

D. Behcet's disease (0 Points)

Your answer: (0 Points)

Question: What is the life stage of Toxoplasma gondii associated with reactivated disease? (1
Points)
Answer:

A. Tachyzoite (1 Points)

Answer Feedback:
The form of T. gondii responsible for uveitic disease is the tachyzoite, an asexually dividing
life stage of the parasite. Infection is typically from ingestion of oocytes, or the sexually
reproducing egg form. Bradyzoites are a cyst form of the organism which can persist latent for
many years. Cystozoite is an older term for bradyzoite.

B. Bradyzoite (0 Points)
C. Cystozoite (0 Points)
D. Oocyst (0 Points)

Your answer: (0 Points)

Question: Which of the following is not a correct HLA association of uveitic diseases? (1 Points)
Answer:

A. Birdshot chorioretinopathy = HLA A29 (0 Points)


B. Acute anterior uveitis = HLA B27 (0 Points)
C. Tubulointerstitial nephritis and uveitis syndrome = HLA DRB1 (0 Points)
D. Fuchs heterochromic cyclitis = HLA DR-4 (1 Points)

Answer Feedback:
Birdshot choroiditis (HLA-A29), acute anterior uveitis (HLA-B27), and tubulointerstitial
nephritis and uveitis syndrome (TINU, HLA-DRB1*0102) all have extremely high relative risks
associated with their specific HLA alleles. Fuchs heterochromic cyclitis does not have a
known HLA association.

Your answer: (0 Points)

Print

AAO Uveitis 04 Short

Total Questions: 9 (9 Points)


Correctly Answered Questions: 0 (0 Points)
Percentage Score 0%

Questions

Question: Early findings in Vogt Koyanagi Harada (VKH) syndrome does not include which of the
following? (1 Points)
Answer:

A. Bilateral serous retinal detachments (0 Points)


B. Granulomatous anterior segment inflammation (0 Points)
C. Tinnitus (0 Points)
D. Vitiligo (1 Points)

Answer Feedback:
The prodromal and acute phases of VKH typically include CNS findings such as meningismus
and headache, auditory findings including vertigo, tinnitus, and dysacoustia, and ocular
findings including granulomatous inflammation and serous retinal effusions. Skin findings
such as vitiligo, alopecia, and poliosis tend to be late findings, as is the 'sunset glow' fundus.

Your answer: (0 Points)

Question: Which of the following disorders is not always bilateral? (1 Points)


Answer:

A. Pars planitis (1 Points)

Answer Feedback:
Birdshot chorioretinopathy, VKH, and sympathetic ophthalmia are presumed autoimmune
conditions that always affect both eyes. While 90% of pars planitis cases are bilateral, a
unilateral form of the disease has been described.

B. Sympathetic ophthalmia (0 Points)


C. Birdshot chorioretinopathy (0 Points)
D. Vogt-Koyanagi-Harada disease (0 Points)

Your answer: (0 Points)

Question: The 'white dot' lesions of which disease is least apparent on fluorescein angiography? (1
Points)
Answer:

A. Serpiginous choroiditis (0 Points)


B. Punctate inner choroidopathy (PIC) (0 Points)
C. Acute posterior multifocal placoid pigment epitheliopathy (APMPPE) (0 Points)
D. Birdshot chorioretinopathy (1 Points)

Answer Feedback:
APMPPE, PIC, and serpiginous choroiditis lesions generally show a 'early block, late stain'
pattern on fluorescein angiography. In contrast, birdshot choroiditis lesions tend to be
angiographically silent in the active phases of disease. They may be visualized better with
indocyanin green angiography.

Your answer: (0 Points)

Question: Which of the following drug side-effect relationships is incorrect? (1 Points)


Answer:

A. Methotrexate -- hepatotoxicity (0 Points)


B. Cyclosporine A -- hypertension (0 Points)
C. Azathioprine -- sterility (1 Points)

Answer Feedback:
Azathioprine is associated with bone marrow and hepatic toxicity. Alkylating agents are
associated with sterility. Cyclosporine A side effects include hirsutism, tremor, hypertension,
and renal dysfunction. Infliximab is associated with tuberculosis reactivation, worsening of
multiple sclerosis, and possible increased risk of lymphoma. Methotrexate is associated with
hepatotoxicity and rarely pulmonary fibrosis.

D. Infliximab -- exacerbation of multiple sclerosis (0 Points)

Your answer: (0 Points)

Question: Which of the following uveitic conditions is most likely to present with a white and
asymptomatic eye? (1 Points)
Answer:

A. Juvenile idiopathic arthritis-associated uveitis (1 Points)

Answer Feedback:
Juvenile idiopathic arthritis-associated uveitis typically presents with a white eye and no
symptoms. Patients diagnosed with JIA, particular the pauciarticular, ANA+ variety, must
undergo regular eye examinations. The other conditions here would all typically present with
anterior segment injection, pain, and photophobia.
B. Herpetic anterior uveitis (0 Points)
C. Acute retinal necrosis syndrome (0 Points)
D. Wegener's granulomatosis (0 Points)

Your answer: (0 Points)

Question: A purified protein derivative (PPD) test greater than ____ mm is considered positive in all
individuals. (1 Points)
Answer:

A. 10 mm (0 Points)
B. 2 mm (0 Points)
C. 5 mm (0 Points)
D. 15 mm (1 Points)

Answer Feedback:
The PPD is read 48-72 hours after placement, and induration measured. A reading greater
than 15 mm is considered positive in all individuals. In individuals at risk, including health care
workers or potential contacts with active cases, 10 mm is considered positive. In individuals
with AIDS or positive chest X-ray findings, 5 mm is considered positive. A 2 mm PPD would
be considered negative.

Your answer: (0 Points)

Question: What would the differential diagnosis for isolated intermediate uveitis not include? (1
Points)
Answer:

A. Lyme disease (0 Points)


B. Reactive arthritis (1 Points)

Answer Feedback:
The differential diagnosis for isolated intermediate uveitis includes multiple sclerosis (found in
10-30% of patients with intermediate uveitis), Lyme disease, sarcoidosis, syphilis,
tuberculosis, intraocular lymphoma, and ocular ischemic syndrome. Reactive arthritis is more
typically associated with conjunctivitis and anterior uveitis.

C. Primary CNS lymphoma (0 Points)


D. Multiple sclerosis (0 Points)

Your answer: (0 Points)

Question: The fluocinolone acetonide implant releases therapeutic levels of corticosteroids to the
vitreous cavity for approximately how many days? (1 Points)
Answer:

A. 500 (0 Points)
B. 1000 (1 Points)

Answer Feedback:
The fluocinolone acetonide implant (Retisert) releases therapeutic levels of drug for
approximately 1000 days (~3 years). Reactivation rates for intermediate and posterior uveitis
are much lower in treated eyes than in untreated controls. However, ~90% of phakic patients
will develop cataract with this device and ~40-50% will require glaucoma surgery.

C. 5000 (0 Points)
D. 100% (0 Points)

Your answer: (0 Points)

Question: What is the predominant cell type seen in the anterior chamber of a patient with phacolytic
uveitis and glaucoma? (1 Points)
Answer:

A. Macrophage (1 Points)

Answer Feedback:
Phacolytic glaucoma occurs when lens proteins leech from liquified cortex of a hypermature
cataract into the anterior chamber. Macrophages respond to this protein and engulf it,
becoming 'lipid laden'. These cells clog the trabecular meshwork, leading to glaucoma.
Treatment requires acute corticosteroids, pressure lowering agents, and prompt removal of
the cataract.

B. Neutrophil (0 Points)
C. Eosinophil (0 Points)
D. T-lymphocyte (0 Points)

Your answer: (0 Points)

Print

AAO Uveitis 05 Short

Total Questions: 9 (9 Points)


Correctly Answered Questions: 0 (0 Points)
Percentage Score 0%

Questions

Question: Retinal vasculitis affecting only the veins is most likely to be associated with which
condition? (1 Points)
Answer:

A. Systemic lupus erythematosis (0 Points)


B. Behcet disease (0 Points)
C. Polyarteritis nodosa (0 Points)
D. Ocular sarcoidosis (1 Points)

Answer Feedback:
A pure retinal periphlebitis is most likely to be associated with ocular sarcoidosis. The form of
this inflammation has been called 'candlewax drippings' or Taches de Bougie. Polyarteritis
nodosa is typically associated with arteritis in the eye. Systemic lupus erythematosis and
Behcets disease may present with mixed arterial and venous inflammation.

Your answer: (0 Points)

Question: Which of the following approaches is most appropriate for cataract surgery in a patient with
Fuch's heterochromic iridocyclitis without glaucoma? (1 Points)
Answer:

A. Start methotrexate with a goal of no more than rare cell before surgery (0 Points)
B. Start acyclovir, because Fuch's heterochromic iridocyclitis has been associated with rubella
virus (0 Points)
C. Standard phacoemulsification with a posterior chamber intraocular lens implant (IOL) (1
Points)

Answer Feedback:
Eyes with Fuch's heterochromic iridocyclitis tend to do well with standard cataract surgery
techniques and intraocular lens (IOL) implant placement. Some uveitis specialists will treat
briefly before surgery with topical and even oral corticosteroids, but systemic
immunomodulatory therapy such as one with methotrexate is not usually pursued as one
would pursue for chronic anterior uveitis associated with such diseases as sarcoidosis, VKH
disease, or juvenile rheumatoid arthritis. FHI has been associated with Rubella virus, but
acyclovir is not an effective treatment.

D. Perform a lensectomy and vitrectomy, and leave the eye aphakic to prevent ciliary body
membranes and hypotony (0 Points)

Your answer: (0 Points)

Question: A 47-year-old woman with birdshot chorioretinopathy has 20/40 Snellen visual acuity. She
has received no treatment for this disease. What is the most likely case of decreased Snellen visual
acuity? (1 Points)
Answer:

A. Glaucoma (0 Points)
B. Cystoid macular edema (1 Points)

Answer Feedback:
The most common cause of decreased visual acuity in birdshot chorioretinopathy is cystoid
macular edema. Glaucoma or cataract formation is unusual in birdshot chorioretinopathy
except secondary to corticosteroids, because there is little anterior uveitis. Retinal vasculitis is
common in birdshot chorioretinopathy, but is not commonly associated in itself with
decreased visual acuity.

C. Retinal vasculitis (0 Points)


D. Cataract (0 Points)

Your answer: (0 Points)

Question: A patient with Vogt-Koyanagi-Harada disease was treated with high-dose oral prednisone
and had resolution of her serous retinal detachments. The dose has been gradually decreased, and 4
months into therapy, she is on prednisone 20 mg daily. She is on topical prednislolone acetate 1% QID
and her visual acuity is 20/30 both eyes. There are 2+ cells in the anterior chamber of both eyes with
no vitritis, cystoid macular edema, or subretinal fluid. What is the most appropriate next step? (1
Points)
Answer:
A. Posterior subtenon injections of triamcinolone acetonide (0 Points)
B. Start a corticosteroid-sparing immunomodulatory agent (1 Points)

Answer Feedback:
The goal generally is to have a patient on 10 mg prednisone per day or less by 6 months; if
not, a corticosteroid-sparing immunomodulatory agent is needed to allow further weaning of
the prednisone. This is considered the standard of care. Periocular injections do not tend to
be as effective in the chronic anterior uveitis associated with VKH disease, but could be
considered. The flucinolone implant is not approved for anterior chamber inflammation.

C. Increase the oral prednisone to 40 mg/day (0 Points)


D. Place a flucinolone implant (0 Points)

Your answer: (0 Points)

Question: A 23-year-old man presents complainig of floaters in his left eye for 2 weeks. The visual
acuity is 20/20 in both eyes. There is a mild (1+) anterior chamber and vitreous reaction, with a small
focus of retinitis (less than 1/2 disc diameter) adjacent to a chorio-retinal scar, anterior to the vascular
arcades in the left eye. He is HIV negative. He is allergic to sulfa medication. What is the most
appropriate next step? (1 Points)
Answer:

A. Oral clindamycin, and after 2 days of therapy, start oral prednisone (0 Points)
B. Observe closely without treatment (1 Points)

Answer Feedback:
Not every reactivation of toxoplasmosis retinochoroiditis has to be treated; it is difficult for
immunocompetent patients even to show that treatment has much effect. Certainly, lesions
that are sight-threatening (ie, in the macula or by the optic nerve head) are most often
treated, as well as lesions that are large and associated with significant vitritis (if for no other
reason than to be able to give corticosteroids). If he were HIV positive, particualrly with a low
CD4 count, treatment and evaluation for systemic or central nervous system toxoplasmosis
would be indicated. This could be early viral acute retinal necrosis and close follow-up is
indicated, but the adjacent chorioretinal scar makes that highly unlikely.

C. Hospitalization for intravenous acyclovir (0 Points)


D. Intravitreal clindamycin and dexamethasone (0 Points)

Your answer: (0 Points)

Question: A patient with a bilateral anterior and intermediate uveitis is suspected of having
sarcoidosis. There are no conjunctival or lid granulomata. A chest x-ray shows no abnormalities, and
serum angiotensin converting enzyme is normal. Which one of the following is the most appropriate
examination for pursuing the diagnosis of sarcoidosis? (1 Points)
Answer:

A. Biopsy of the conjunctiva (0 Points)


B. HLA-B27 status (0 Points)
C. Repeat serum angiotensin converting enzyme to rule out laboratory error (0 Points)
D. High-resolution CT scan of the chest (1 Points)

Answer Feedback:
The most likely diagnosis is sarcoidosos. Both chest x-ray and angiotensin-converting
enzymes are often used to screen for sarcoidosis, but they are neither very sensitive nor
specific. Biopsy of the conjunctiva, unless there are suspicious lesions, has a very low yield.
Your answer: (0 Points)

Question: A 6-year-old child presents with intermediate uveitis and a peripheral retinal granuloma in
one eye. What is the most appropriate diagnostic test? (1 Points)
Answer:

A. Serum herpes simplex titers (0 Points)


B. A complete blood count (CBC) with differential (0 Points)
C. Serum toxocara titers (1 Points)

Answer Feedback:
The age of the patient and presentation are most consistent with toxocara. A complete blood
count could show abnormalities in various forms of uveitis, and possibly eosinophilia in
toxocara, but other than in leukemia or systemic lymphoma, it is not likely to lead to a specific
diagnosis. This presentation is not typical of HLA-B27 disease and is postiive in 5-10% of the
normal population, so even if the child is HLA-B27 positive, it would not be helpful. Similarly,
herpes titers are postivie in the majority of the population and a granuloma is not typical of
herpetic disease.

D. HLA-B27 status (0 Points)

Your answer: (0 Points)

Question: Which factor is most important in diagnosing Behcet's disease? (1 Points)


Answer:

A. Findings of ischemia on indocyanine green angiography (0 Points)


B. HLA-B51 status (0 Points)
C. A systemic history and physical examination (1 Points)

Answer Feedback:
While the diagnositc criteria for Behcet's disease include uveitis consistent with the diagnosis,
that is not sufficient to make the diagnosis. Although a hypopyon can be associated with
Behcet's disease, it is neither sensitive nor specific for the diagnosis; i.e., most eyes with
hypopyon do not have Behcet's disease, and many patients with Behcet's disease do not
have hypopyon. Similarly, ischemia on the indocyanine green test is not specific for Behcet's.
Much more sensitive (although not necessarily specific) are the major criteria: presence of
oral or genital ulcers, or skin lesions, along with ocular inflammation. The minor criteria are
arthritis, ulceration of the bowel, epididymitis, vasculitis, and neuropsychiatric symptoms.
HLA-B51 has a somewhat low relative risk for Behcet's disease and is not part of the
diagnostic criteria; while it is a reasonable test to obtain, it is not sufficient or necessary for
the diagnosis.

D. The presence of a hypopyon on slit lamp examination (0 Points)

Your answer: (0 Points)

Question: Which one of the following tests is most likely to give a definitive diagnosis in a
immunocompetent patient with a rapidly progressing peripheral retinal necrosis in one eye? (1 Points)
Answer:

A. Toxoplasma serologies (0 Points)


B. Vitreous for polymerase chain reaction testing for herpes viruses (1 Points)

Answer Feedback:
Most acute retinal necrosis in immunocompromised individuals will be from herpes viruses,
either simplex or zoster. Putative auto-immune retinitis (such as Behcets disease), neoplasia
(such as lymphomas), and other infections (such as cytomegalovirus or toxoplasmosis), can
rarely give a similar picture. Toxoplamsosis titers are reasonable, but not as likely to give a
definitive diagnosis; if negative they are reassuring (although there can be false negatives),
but if positive, they do not usually give a definitive diagnosis, because they mean only that the
patient has had toxoplasmosis, but not that it is causing ocular disease. This is also a very
atypical presentation for toxoplasmosis in an immunocompetent patient (although in AIDS
patients toxoplasmosis can mimic CMV retinitis). A complete blood count is reasonable to
obtain, because it could reflect a systemic leukemia or lymphoma, but that is again much less
common, and if intraocular, may not be systemic. Herpes serologies are almost worthless,
because the vast majority of the population has positive serologies, and herpes serologies will
not help determine the cause of the retinitis.

C. Herpes serologies (0 Points)


D. Complete blood cell count with differential (0 Points)

Your answer: (0 Points)

Print

AAO Uveitis 06 Short

Total Questions: 9 (9 Points)


Correctly Answered Questions: 0 (0 Points)
Percentage Score 0%

Questions

Question: Which of the following clinical features are more common in scleritis than episcleritis? (1
Points)
Answer:

A. It is simple or nodular, and is not associated with systemic collagen-vascular disease (0


Points)
B. It blanches in response to topical phenylephrin, while episcleritis does not (0 Points)
C. It is potentially progressively destructive and sight threatening if left untreated (1 Points)

Answer Feedback:
Both episcleritis and scleritis are associated with collagen-vascular diseases, among other
etiologies. Scleritis is generally progressive if left untreated. It typically features deep,
violaceous redness and tenderness to palpation. Scleritis may be nodular or diffuse.
Episcleritis, but not scleritis, typically blanches in response to topical phenylephrin.

D. Minimal tenderness compared with episcleritis (0 Points)

Your answer: (0 Points)


Question: Which of the following tests is the most sensitive mode of work-up for a patient with the
presentation associated with chronic epistaxis and sinusitis in the attached image? (1 Points)
Answer:

A. Scleral biopsy (0 Points)


B. Magnetic resonance imaging (MRI) of orbits (0 Points)
C. Scleral culture (0 Points)
D. Serology (1 Points)

Answer Feedback:
Necrotizing scleritis is a destructive type of scleritis that is associated with systemic vasculitic
diseases, such as Wegener's granulomatosis. Serological assay for c-ANCA, or anti-
neutrophil cytoplasmic antibody, is a highly sensitive test to confirm the diagnosis of WG. If
positive, subsequent testing for myeloperoxidase and/or proteinase-3 is generally performed.
Serology is more sensitive than orbital imaging and scleral biopsy.

Your answer: (0 Points)

Question: Most patients presenting with acute anterior nongranulomatous anterior uveitis will require
what treatment? (1 Points)
Answer:

A. Oral corticosteroids (0 Points)


B. Periocular corticosteroids injections (0 Points)
C. Immunosuppressive agents orally (0 Points)
D. Topical corticosteroids and cycloplegic agents only (1 Points)

Answer Feedback:
Most acute anterior uveitis responds well to topical therapy. In recalitrant cases, addition of
oral corticosteroids can hasten resolution, and have the advantage of being rapidly tapered if
the patient is intolerant. Corticosteroid monotherapy is relatively contraindicated in herpes
virus-associated uveitis. Oral antiviral medication should be utilized in conjunction with local
and systemic corticosteroids in this case.

Your answer: (0 Points)

Question: Which of the following patients is most likely to have scleritis? (1 Points)
Answer:

A. A 30-year-old woman with red eye and blurred vision (0 Points)


B. A 40-year-old male with red eye and throbbing eye pain with weight loss and non-healing skin
lesions (1 Points)

Answer Feedback:
Scleritis is often associated with systemic vasculities, including polyarteritis nodosum,
Wegener's granulomatosis, Churg-Strauss syndrome, and systemic lupus erythematosis.
Weight loss and skin lesions would be consistent with a systemic vasculitis.

C. A 73-year-old male with red eye and decreased vision with ipsilateral headaches and jaw
pain (0 Points)
D. A 56-year-old woman with foreign body sensation and red eye (0 Points)

Your answer: (0 Points)

Question: Which of the following patients is most likely to have primary central nervous
system/intraocular lymphoma? (1 Points)
Answer:
A. A 40-year-old male with cotton wool spots and hard exudates (0 Points)
B. A 59-year-old male with hemorrhagic retinitis and retinal vasculitis (0 Points)
C. A 65-year-old female with dense vitritis, subretinal infiltrates, and mental confusion (1 Points)

Answer Feedback:
Subretinal infiltrates and mental confusion are typical for primary central nervous
system/intraocular lymphoma. The patient's age is also compatible.

D. A 29-year-old female with pars plana exudates and retinal vasculitis (0 Points)

Your answer: (0 Points)

Question: Which of the following statements best describes the epidemiology of scleritis associated
with collagen vascular disease? (1 Points)
Answer:

A. About half of the patients have an underlying systemic medical condition. (1 Points)

Answer Feedback:
About 40 to 50 percent of patients with scleritis will have an underlying collagen vascular
disease or infectious etiology, most commonly rheumatoid arthritis.

B. It is most common in third decade. (0 Points)


C. Bilateral involvement is seen only rarely. (0 Points)
D. Men are more commonly affected than women. (0 Points)

Your answer: (0 Points)

Question: Which of the following statements best describes the demographics of acute posterior
multifocal placoid epitheliopathy? (1 Points)
Answer:

A. The male to female ratio is greater than 1. (0 Points)


B. It has a seasonal predilection. (0 Points)
C. It is a disease that affects caucasians only. (0 Points)
D. Young, healthy adults are frequently affected. (1 Points)

Answer Feedback:
AMPPE tends to be a disease of young, healthy individuals. It may have a viral prodrome.
Unlike punctate inner choroiditis, it does not have a female prediliction.

Your answer: (0 Points)

Question: Which of the following is a complication of punctate inner choroidopathy? (1 Points)


Answer:

A. Optic atrophy (0 Points)


B. Cystoid macular edema (0 Points)
C. Choroidal neovascularization (1 Points)

Answer Feedback:
Up to 40% of patients with punctate inner choroidopathy (PIC) will develop choroidal
neovascularization (CNV). Inflammation in PIC is limited to the inner choroid, the vitreous is
generally without inflammation, and other complications are very rare with this condition.

D. Retinal detachment (0 Points)

Your answer: (0 Points)

Question: Which one of the following tests is most important for patients with either intermediate or
posterior uveitis, but without keratitis or scleritis? (1 Points)
Answer:

A. HLA-B27 (0 Points)
B. Serum rheumatoid factor (0 Points)
C. Serum antinuclear antibodies (0 Points)
D. Treponema-specific syphilis serologies (1 Points)

Answer Feedback:
It is always important to rule out syphilis, a treatable disease using anti-treponemal specific
testing. HLA-B27 is rarely associated with posterior uveitis without a significant anterior
uveitis, but is most often associated with recurrent unilateral acute anterior uveitis. In any
case, a positive HLA-B27 test does not make a specific diagnosis, as almost 10% of the
Caucasian population is HLA-B27 positive, so it would be little help here. Similarly, anti-
nuclear antibodies can be found in normal individuals. Neither rheumatoid arthritis (RA) nor
systemic lupus erythematosis (SLE) is commonly associated with uveitis without scleritis or
retinal vasculitis. If either are suspected, a good history and physical exam are more
important than these serologies.

Your answer: (0 Points)

Print

AAO Uveitis 07 Short

Total Questions: 9 (9 Points)


Correctly Answered Questions: 0 (0 Points)
Percentage Score 0%

Questions

Question: Mycophenolate mofetil is not appropriate for treatment of which of the following
conditions? (1 Points)
Answer:

A. Fuchs’ iridocyclitis syndrome (1 Points)

Answer Feedback:
Anti-inflammatory therapy is not generally necessary for Fuchs’ iridocyclitis syndrome.
Mycophenolate mofetil is commonly used as part of long-term, steroid-sparing,
immunosuppressive therapy for the other conditions listed. It should be noted that this is an
off-label use of this agent.

B. Birdshot retinochoroidopathy (0 Points)


C. Sympathetic ophthalmia (0 Points)
D. Rheumatoid arthritis-associated scleritis (0 Points)

Your answer: (0 Points)

Question: A 39-year-old man with AIDS and cytomegalovirus (CMV) retinitis in 1 eye undergoes
successful placement of an intravitreal ganciclovir implant without systemic anti-CMV therapy. He
declines highly active antiretroviral therapy. After 1 month, the retinitis is inactive. Nine months after
surgery, the retinitis is active, and the patient's CD4+ count is 20 cells/µL. He feels well in general, and
the opposite eye shows no retinitis. What is the most appropriate approach at this time? (1 Points)
Answer:

A. Place another ganciclovir implant (1 Points)

Answer Feedback:
This patient demonstrates recurrence of retinitis following exhaustion of the drug in the
ganciclovir implant, which usually provides active drug delivery for 7-8 months. Because the
patient’s CD4+ count remains low, additional anti-CMV therapy is indicated. The first implant
can be left in place and another implant placed in a different quadrant. Intravenous foscarnet
requires placement of a permanent indwelling catheter for daily infusions and is a less
preferable approach. Acyclovir is not effective for the treatment of CMV retinitis. It is worth
noting that mortality rates are higher for patients with CMV retinitis who are not placed on
systemic anti-CMV therapy; for this reason and for fellow eye prophylaxis, it would also be
desirable to place the patient on oral valganciclovir.

B. Make no changes in therapy (0 Points)


C. Begin intravenous foscarnet (0 Points)
D. Begin oral acyclovir (0 Points)

Your answer: (0 Points)

Question: A 39-year-old man with AIDS and cytomegalovirus (CMV) retinitis in 1 eye is treated with
valganciclovir, but declines highly active antiretroviral therapy. After 2 months, the retinitis is inactive.
Nine months after beginning therapy, the retinitis is active, and the patient's CD4+ count is 15 cells/µL.
He undergoes placement of a ganciclovir implant. Two months later, the retinitis remains active. What
would the most appropriate approach at this time be? (1 Points)
Answer:

A. Place another ganciclovir implant (0 Points)


B. Allow more time for the ganciclovir implant to take effect (0 Points)
C. Begin weekly intravitreous ganciclovir injections (0 Points)
D. Begin intravenous foscarnet (1 Points)

Answer Feedback:
The patient has likely developed ganciclovir-resistant CMV retinitis. Placing another
ganciclovir implant, or giving intravitreous ganciclovir injections, is unlikely to be effective.
Ganciclovir-resistant CMV retinitis may respond to intravenous foscarnet.

Your answer: (0 Points)

Question: Which of the following statements best describes the typical fundus findings of serpiginous
choroiditis? (1 Points)
Answer:
A. Intense vitritis is common. (0 Points)
B. Multiple, new, isolated lesions occur with recurrences. (0 Points)
C. Old, scarred lesions may be present in the newly-diagnosed eye. (1 Points)

Answer Feedback:
Serpiginous choroiditis is a typically recurrent, bilateral disease that can lead to significant
vision loss if untreated. Inflammation of the choriocapillaris leads to loss, which is typically by
contiguous spread, although new foci can appear. Minimal, if any, vitritis is present, and
retinal vasculitis is not seen. Treatment is with immunomodulatory drugs.

D. Vasculitis is a prominent feature. (0 Points)

Your answer: (0 Points)

Question: In which of the disorders below is indocyanine green (ICG) angiography most useful? (1
Points)
Answer:

A. Pars planitis (0 Points)


B. Posterior scleritis (0 Points)
C. Primary central nervous system/intraocular lymphoma (0 Points)
D. Serpiginous choroiditis (1 Points)

Answer Feedback:
ICG angiography is occasionally helpful in making the diagnosis of an individual with posterior
uveitis. It can be used, for example, in distinguishing macular serpiginous choroiditis from
persistent placoid maculopathy. The former will show resolution of the macular
hypofluorescence, while the latter will show persistent hypofluorescence. That said, the utility
of ICG angiography is generally less than fluorescein angiography, as it does not show
aspects of retinal circulation or disease.

Your answer: (0 Points)

Question: Which of the following is a recommended treatment for serpiginous choroiditis involving the
macula? (1 Points)
Answer:

A. Alkylating agents (1 Points)

Answer Feedback:
Vision threatening macular serpiginous choroiditis requires prompt institution of strong
immunomodulatory therapy. Success has been reported for alkylating agents and for use of
transplant-dose therapy with cyclosporine A, azathioprine, and prednisone. Corticosteroids
alone are often insufficient to treat this condition. While there has been some discussion
suggesting a herpetic genesis for serpiginous choroiditis, definitive evidence has not been
demonstrated, and acyclovir should be used adjunctively if at all. Topical corticosteroids have
no effect on this condition.

B. Oral corticosteroids (0 Points)


C. Topical corticosteroids (0 Points)
D. Acyclovir (0 Points)

Your answer: (0 Points)

Question: A 25-year-old, Brazilian man presents with a history of decreased vision in his left eye for 1
week. Visual acuity is 20/70, and moderate vitreitis is present. On dilated examination, a pigmented
scar in the posterior pole with adjacent yellow white chorioretinitis is present. What is the most
appropriate treatment? (1 Points)
Answer:

A. Oral corticosteroids (0 Points)


B. Pyrimethamine, sulfadiazine, and folinic acid (1 Points)

Answer Feedback:
This patient likely has ocular toxoplasmosis. While most ocular toxoplasmosis is thought to be
reactivation, and will be accompanied by a chorioretinal scar, recent work has suggested that
de novo infection can also be associated with uveitis, particularly in endemic areas, such as
southern Brazil. Appropriate treatments for ocular toxoplasmosis include combination
pyrimethamine, sulfadiazene, folinic acid; trimethoprim-sulfamethoxazole; azithromycin; and
clindamycin (or combinations thereof). Amphotericin B is an antifungal agent; IV acyclovir is
more appropriate for acute retinal necrosis due to herpes virus; and oral corticosteroids
without antibiotic use are contraindicated with active ocular toxoplasmosis.

C. IV acyclovir (0 Points)
D. Amphotericin B (0 Points)

Your answer: (0 Points)

Question: A 5-year-old patient with juvenile chronic arthritis and chronic anterior uveitis was started
on methotrexate and achieved control of the ocular inflammation for the first time in 6 months of
therapy. However, the child now complains of abdominal pains and refuses to take the medicine.
Which approach would most likely succeed in this setting? (1 Points)
Answer:

A. Have the parent administer the drug by subcutaneous injection (1 Points)

Answer Feedback:
Use of corticosteroids carries significant morbidity for pediatric patients, including growth
retardation, cushingoid changes, and endocrine abnormalities. Methotrexate has long been
used by rheumatologists as a steroid sparing agent for juvenile arthritis, and has a good
safety profile in this population. Subcutaneous injection of methotrexate may alleviate
gastrointestinal side effects of the drug. It also may improve compliance and offer slightly
improved efficacy as first-pass hepatic metabolism is bypassed. The drug is given weekly,
usually at a dose of 7.5 mg - 20 mg, titrated to effect. Many rheumatologists also favor use of
folic acid (600 - 800 mcg daily), which may decrease side effects of methotrexate.

B. Mix the methotrexate pills with a favorite food (0 Points)


C. Administer the methotrexate pills at bedtime, so the child can sleep through the period of
discomfort (0 Points)
D. Switch to an alternative immunosuppressive agent (0 Points)

Your answer: (0 Points)

Question: Which of the following is the most appropriate treatment for a 30-year-old man with sinus
inflammation, bilateral scleritis, mild anterior uveitis, and a positive anti-neutrophilic cytoplasmic
antibodies (c-ANCA) test? (1 Points)
Answer:

A. Topical non-steroidal anti-inflammatory agents such as ketralac (0 Points)


B. Oral non-steroidal, anti-inflammatory agents such as indomethacin at 50 mg tid (0 Points)
C. Injection of 40 mg of periocular triamcinolone actetate (0 Points)
D. Oral cyclophosphamide at 2 mg/kg/day (1 Points)

Answer Feedback:
Several ocular inflammatory conditions mandate initial treatment with strong
immunomodulation. These conditions include Wegeners granulomatosis, serpiginous
choroiditis, Behcet disease, and mucous membrane pemphigoid. Treatment options for these
conditions include alkylator therapy or transplant-level immunomodulation with cyclosporine,
azathioprine, and prednisone.

Your answer: (0 Points)

Print

AAO Uveitis 08 Short

Total Questions: 10 (10 Points)


Correctly Answered Questions: 0 (0 Points)
Percentage Score 0%

Questions

Question: What do symptoms of serpiginous choroidopathy include? (1 Points)


Answer:

A. Redness (0 Points)
B. Scotoma (1 Points)

Answer Feedback:
Serpiginous choroiditis causes acute inflammation of the choroid and choriorcapillaris, with
subsequent loss of retinal pigment epithelium and photoreceptors. The resulting retinal
dysfunction manifests as scotoma. Pain, photophobia, and redness are typically associated
with anterior uveitis.

C. Photophobia (0 Points)
D. Pain (0 Points)

Your answer: (0 Points)

Question: A patient is being evaluated by a rheumatologist for the possibility of sarcoidosis. While the
patient has no ocular complaints and has a normal eye exam, the rheumatologist requests that a
conjunctival biopsy be performed to help confirm the diagnosis of sarcoidosis. Which of the following is
the most appropriate response to the rheumatologist? (1 Points)
Answer:

A. We should rely on laboratory testing, such as angiotensin converting enzyme or gallium scan,
instead of biopsy. (0 Points)
B. There are no alternatives to the conjunctiva in making a tissue diagnosis of sarcoidosis. (0
Points)
C. Blind conjunctival biopsies without a visible lesion will have a low diagnostic yield. (1 Points)

Answer Feedback:
Sarcoidosis is a tissue diagnosis, and efforts should be made to identify involved tissues for
biopsy. If conjunctival granulomas are observed, they may be biopsied with high diagnostic
yield. In the absence of visible conjunctival lesions, however, biopsy yields are less than 10%.
Higher yield may be obtained by chest CT scanning followed by mediastinoscopy or
bronchoscopy. However, such invasive diagnostic tests should be considered only when
making the sarcoidosis diagnosis has systemic treatment implications for the patient.

D. Conjunctival biopsy is contraindicated in all patients with sarcoidosis. (0 Points)

Your answer: (0 Points)

Question: A 34-year-old man has a localized area of scleral thinning 5 mm nasal to the limbus. The
surrounding episcleral and conjunctival tissue is not injected. The patient had eye surgery four years
before presentation. He is healthy without any systemic inflammatory or infectious disease. What is the
most likely surgery he had that could result in this finding? (1 Points)
Answer:

A. Pterygium excision with mitomycin C or Beta-irradiation (1 Points)

Answer Feedback:
Necrotizing scleritis certainly raises concerns about systemic disease, but one must consider
other causes (foreign bodies, infection, previous mitomycin or radiation). The nasal position
would make a trabeculectomy with mitomycin unlikely, but is a typical location for pterygia.

B. Trabeculectomy (0 Points)
C. LASIK (0 Points)
D. Laser retinopexy (0 Points)

Your answer: (0 Points)

Question: A 64-year-old Caucasian woman presents with a history of floaters and mildly blurred vision
for two weeks in her left eye. On examination of the right eye there is no inflammation. The left eye has
mild anterior uveitis, moderate vitritis, and a localized area of retinitis of about one half disc diameter in
size nasal to the optic nerve head, but posterior to the equator of the globe. The optic nerve head is not
involved. She is otherwise healthy with no known systemic disease and is on no medications. What is
the most likely diagnosis? (1 Points)
Answer:

A. Toxoplasmosis (1 Points)

Answer Feedback:
Toxoplasmosis is the most common cause of unilateral retinitis in this population. Toxocara
would be unlikely in this age group. Acute retinal necrosis is most often posterior to the
equator, and after two weeks would have likely enlarged ion a circumfrential fashion. Behcets
disease is unlikely in a middle aged Caucasian woman without other findings and is certainly
less likely than toxoplasmosis.

B. Behcets disease (0 Points)


C. Toxocara (0 Points)
D. Acute retinal necrosis (0 Points)

Your answer: (0 Points)


Question: A 35-year-old man has decreased vision, pain, redness, and photophobia in the left eye of
three days duration. He is otherwise healthy, without known systemic disease. On examination the right
eye is without inflammation. Examination of the left eye reveals 3+ anterior chamber cells and 2-3+
vitreous cells, and haze with 5 clock hours of retinitis anterior to the equator. He is HIV negative. What
is the most likely diagnosis? (1 Points)
Answer:

A. toxoplasmosis (0 Points)
B. Sarcoidosis (0 Points)
C. Toxocara (0 Points)
D. Viral acute retinal necrosis (1 Points)

Answer Feedback:
A rapid onset of symptomatic retinitis anterior to the equator in a non-immunosuppressed
patient best fits the description of viral acute retinal necrosis. Toxocara would be an unusual
cause or retinal necrosis, especially in this age group. Sarcoidosis is not the differential
diagnosis, but is less likely with this picture. A bilateral granulomatous pan uveitis, not quite
as rapid in its course, would be much more typical, although retinal necrosis is possible.
Toxoplasmosis is unlikely to result in such a large area of retinal necrosis unless there is
immunosuppression, including corticosteroid injection.

Your answer: (0 Points)

Question: A 28-year-old Hispanic woman presents with three days of headache and loss of vision in
both eyes. She was admitted for suspicion of meningitis, although she has no fever. There was a mild
lymphocytic response noted in her cerebrospinal fluid; stains for microbes were negative, and cultures
are negative to date. On examination she has a moderate bilateral anterior uveitis and vitritis with
serous retinal detachments. There is no retinal necrosis. What is the most appropriate therapy? (1
Points)
Answer:

A. Intravitreal acyclovir (0 Points)


B. Cyclophospahmide (0 Points)
C. Systemic corticosteroids (1 Points)

Answer Feedback:
This is a typical presentation of Vogt-Koyanagi-Harada disease, for which the treatment is
high-dose systemic corticosteroids.

D. Await viral cultures of the cerebrospinal fluid before starting treatment (0 Points)

Your answer: (0 Points)

Question: What is the mechanism of action of infliximab? (1 Points)


Answer:

A. Antimetabolite (0 Points)
B. Anti-tumor necrosis factor antibody (1 Points)

Answer Feedback:
Infliximab is a murine antibody to tumor necrosis factor (TNF) that has been effective in the
treatment of uveitis. It may be used with antimetobolites, both for increased effectiveness and
to try to prevent anti-infliximab antibodies, although this is not always done. Other TNF agents
include adalimumab (a humanized anti-TNF antibody) and etanercept (a pseudoreceptor).
There is less experience with adalimummab, but it may not be as effective as infliximab in
treating uveitis; etanercept certainly is not.

C. Alkylating agent (0 Points)


D. Corticosteroid (0 Points)

Your answer: (0 Points)

Question: What infectious agent has been associated with corneal endothelial and subendothelial
changes, leading to corneal edema in an eye with recurrent unilateral anterior uveitis, in a patient who
is not immunosuppressed? (1 Points)
Answer:

A. Bartonella (0 Points)
B. toxoplasmosis (0 Points)
C. Cytomegalovirus (1 Points)

Answer Feedback:
Cytomegallovirus anterior uveitis has been shown by polymerase chain reaction techniques
to be associated with anterior uveitis, often with endothelial changes and corneal edema that
can recur in corneal grafts, in non-immunosuppressed patients. Treatment is problematic,
because valganciclovir is much more toxic than acyclovir or other agents that are effective for
herpes zoster or herpes simplex. Rubella has been associated with Fuchs heterochromic
iridocyclitis. Bartonella and toxoplasmosis ocular disease rarely, if ever, would present with an
isolated unilateral anterior uveitis leading to corneal edema.

D. Rubella (0 Points)

Your answer: (0 Points)

Question: What infectious agent has been associated with Fuchs heterochromic iridocyclitis? (1
Points)
Answer:

A. Herpes zoster (0 Points)


B. Syphilis (0 Points)
C. Herpes simplex (0 Points)
D. Rubella (1 Points)

Answer Feedback:
Both epidemiologic and polymerase chain reaction studies suggest rubella is associated with
Fuchs heterochromic iridocyclitis. This is problematic, because we do not have good agents
to treat rubella virus infections.

Your answer: (0 Points)

Question: A 14-year-old girl complains of a sudden onset of bilateral eye pain and photophobia. On
ocular examination she has bilateral anterior uveitis. She has had a fever for two weeks, modest flank
and abdominal pain, and weight loss of 10 pounds. Laboratory testing reveals an elevated erythrocyte
sedimentation rate at 80 mm/hour, mild elevation of the serum creatinine, and mild proteinuria. What is
the most likely diagnosis? (1 Points)
Answer:

A. Toxocara uveitis (0 Points)


B. toxoplasmosis (0 Points)
C. Tubulointerstitial nephritis and uveitis (1 Points)
Answer Feedback:
This case illustrates many of the common features of this rare disease. In fact, there is
evidence that the diagnosis is often missed, and that Tubulointersitital nephritis and uveitis
may not be as rare as we have thought in the past. Toxacara uveitis is not bilateral and not
usually associated with systemic disease. Lyme disease can be considered, but the sudden
onset of illness without sings (such as a typical rash), or history consistent with lyme disease
(or the more typical intermediate uveitis), makes it less likely. Toxoplasmosis does not cause
a bilateral anterior uveitis, and systemic toxoplasmosis would be very uncommon in this age
group.

D. Lyme disease (0 Points)

Your answer: (0 Points)

Print

AAO Uveitis 09 Short

Total Questions: 10 (10 Points)


Correctly Answered Questions: 0 (0 Points)
Percentage Score 0%

Questions

Question: What is the most common ocular complication of rheumatoid arthritis? (1 Points)
Answer:

A. Keratoconjunctivitis sicca (1 Points)

Answer Feedback:
Dry eye, in the form of keratoconjunctivitis sicca is by far the most common complication of
rheumatoid arthritis, and it can have devastating consequences to the ocular surface and
visual function. Scleritis and peripheral ulcerative keratitis may destroy the eye if not treated
aggressively. Their presence is indicative of systemic disease, but these findings are not as
common as k. sicca. Anterior uveitis is usually seen associated with the presence of scleritis
or other inflammatory findings.

B. Episcleritis (0 Points)
C. Scleritis (0 Points)
D. Recurrent anterior uveitis (0 Points)

Your answer: (0 Points)

Question: A 33-year-old male presents with a first episode of moderate (2+) anterior chamber cells
and flare, and 3 clock hours of posterior synechiae. Ophthalmoscopy demonstrates 2+ vitreous haze
and scattered, chorioretinal lesions. How would this uveitis be characterized? (1 Points)
Answer:

A. Intermediate and posterior uveitis (0 Points)


B. Panuveitis (1 Points)

Answer Feedback:
The presence of uniform severity of inflammation of the anterior segment, vitreous, and
choroid would qualify as a panuveitis. In cases in which inflammation of one site, such as the
posterior segment, result in spill-over inflammation in other ocular compartments or tissues, a
panuveitis may be mis-classified.

C. Anterior and posterior uveitis (0 Points)


D. Posterior uveitis (0 Points)

Your answer: (0 Points)

Question: A patient with deep ocular pain and subretinal fluid is suspected of having posterior scleritis
with an exudative retinal detachment. What finding would confirm a diagnosis of posterior scleritis? (1
Points)
Answer:

A. A T sign on B-scan ultrasound (1 Points)

Answer Feedback:
Posterior scleritis may be difficult to diagnose because of its occult location. Although
posterior scleritis can be associated with intraocular inflammation, vitritis, and cystoid macular
edema, such findings are non-specific. Fluid exudation between the sclera and posterior
Tenon's capsule near the optic nerve, detectable by echography or MRI, may show the highly
characteristic T-sign.

B. Cystoid macular edema (0 Points)


C. Vitritis (0 Points)
D. Vitreous hemorrhage (0 Points)

Your answer: (0 Points)

Question: A patient has bilateral diffuse non-necrotizing anterior scleritis. His vision is 20/20 OU and
he has normal intraocular pressure. He has no intraocular inflammation, and his evaluation for a
systemic disorder is not revealing. Which of the following interventions is the preferred first line
therapy? (1 Points)
Answer:

A. Topical steroids (0 Points)


B. Intraveous cyclophosphamide (0 Points)
C. Oral nonsteroidal anti-inflammatory drugs (1 Points)

Answer Feedback:
Oral nonsteroidal anti-inflammatory drugs are effective first line therapy for minimally
symptomatic scleritis. If these agents fail to suppress the inflammation, oral corticosteroids
may be considered. Subtenons triamcinolone (STK) can be used to treat non-necrotic
scleritis, however, because of the risk of inducing a scleral melt, STK use is controversial.
Topical corticosteroids have littel effect on scleritis. A flucinolone implant is effective for
posterior uveitis, but is not useful for scleritis. Cyclosporin can be administered orally or
topically, but not given intravenously for ocular disease.
D. Flucinolone implant (0 Points)

Your answer: (0 Points)

Question: A 37-year-old man with prior anterior uveitis has progressively increased inflammation each
time steroid eyedrops are stopped. He had an exacerbation a week ago, and the inflammation has
largely resolved on a tapering schedule of topical corticosteroids. He has developed a posterior
subcapsular cataract and posterior synechiae right eye. Because of recently reduced vision to 20/80 he
wants cataract surgery as soon as possible. What approach to cataract surgery would be preferred?
(1 Points)
Answer:

A. Begin pre-operative oral corticosteroids 3 days prior to prompt cataract surgery (0 Points)
B. Initate methotrexate 1 month before prompt cataract surgery (0 Points)
C. Perform prompt cataract surgery and treat eny post-operative inflammation with aggresive
topical corticosteroid (0 Points)
D. Delay cataract surgery until regimen stable for 3 months (1 Points)

Answer Feedback:
The widely accepted recommendation is to control the intraocular inflammation for at least 3
months before performing elective surgery. Achieving suppression of uveitis may require
immunosuppressives such as methotrexate, but these typically require months to reach their
maximum and stable level of effectiveness. In cases in which inflammation cannot be
suppressed to anterior chamber accelularity, perioperative corticosteroids can be considered
as adjunctive therapy, but with increased risk of perioperative uveitis exacerbation.

Your answer: (0 Points)

Question: Cyclosporine is thought to reduce chronic noninfectious intraocular inflammation by


reducing the activity of which cell type? (1 Points)
Answer:

A. Macrophages (0 Points)
B. B-lymphocytes (0 Points)
C. Langerhans cells (0 Points)
D. T-lymphocytes (1 Points)

Answer Feedback:
Cyclosporing primarily affects T-lymphocytes. Rituximab has an antibody to an antigen
expressed on B-lymphocytes, but may be effective in diseases in which T-lymphocytes play a
role. Langerhans cells are antigen-presenting cells in the skin and ocular surface that are
involved in initiating the inflammatory response. Macrophages are involved in execution of the
inflammatory arc, but they are not directly modulated by cyclosporin.

Your answer: (0 Points)

Question: Which of the following patients would have the highest likelihood of developing posterior
capsular opacification following uncomplicated cataract extraction with posterior chamber lens
implantation? (1 Points)
Answer:

A. A 35-year-old man with intermediate uveitis (1 Points)

Answer Feedback:
Intermediate uveitis is associated with the development of posterior capsular cararact, and
following cataract removal an increased incidenct of opacification of the posterior
capsule. Chorioretinitis from toxoplasmosis is not associated with cataract or posterior
capsular opacification. Oral steroid therapy for rheumatoid arthritis may lead to posterior
capsular cataract. Traumatic cataract may result in capsular or zonular weakness, or
posterior capsular fibrosis, but does not increase the likelihood of post-surgical development
of posterior capsular opacification.

B. A 58-year-old woman with rheumatoid arthritis (0 Points)


C. A 67-year-old man with recurrent toxoplasmosis (0 Points)
D. A 61-year-old man with a traumatic cataract (0 Points)

Your answer: (0 Points)

Question: Which of the following initial therapies would be preferred for macula or optic nerve
threatening acute retinal necrosis? (1 Points)
Answer:

A. Oral valacyclovir (0 Points)


B. Oral acyclovir (0 Points)
C. Intravenous acyclovir (1 Points)

Answer Feedback:
Intravenous acyclovir reaches near immediate virostatic levels and has excellent ocular
penetration. Many uveitis specialists now also advise adjunctive intravitreal antiviral
(foscarnet or ganciclovir). Oral valacyclovir achieves high serum levels of drug, but the delay
to achieve therapeutic levels for macular or optic nerve threatening ARN may be
unacceptable. Oral acyclovir is irregularly absorbed, but should not be relied upon for
immenently threatening disease. Ganciclovir has activity against herpes simplex and zoster,
but is not a first line treatment.

D. Intravenous ganciclovir (0 Points)

Your answer: (0 Points)

Question: Which of the following tests has higher predictive value for Cat Scratch Disease? (1
Points)
Answer:

A. Fluorescent treponemal antibody absorption (FTA-ABS) (0 Points)


B. Enzyme-linked immunosorbent assay (ELISA) for Borrelia burgdorferi IgM and IgG antibodies
followed by a more specific Western blot test (0 Points)
C. Kveim-Siltzbach test (0 Points)
D. Indirect immunofluorescence assay (IFA) for Bartonella immunoglobulin G (IgG) and
Bartonella immunoglobulin M (IgM), followed by re-testing in 10-14 days (1 Points)

Answer Feedback:

Bartonella henselae is the etiologic agent in most cases of Cat Scratch Disease. Two
serologic methods, the indirect fluorescence assay (IFA) and enzyme immunosorbent assay
(EIA) have been evaluated for diagnosis. Commercially available assays use the IFA test. In
general, IFA IgG titers 1:256 strongly suggest active or recent infection. A positive IgM test
strongly suggests acute disease (or very recent infection). The culture of B. henselae is a
fastidious, slow-growing, gram-negative bacterium that requires specific laboratory conditions
for optimal growth. Polymerase chain reaction (PCR) based tests for Bartonella have become
commercially available. These PCR tests have high specificity and can distinguish among the
different Bartonella species, but sensitivity has not been optimal, ranging from 43 to 76
percent.

The FTA-ABS is positive for Bartonella, but is also positive for other treponemal diseases,
such as syphilis. Borrelia burgdorferi causes Lyme disease, not cat scratch disease. The
Kveim test, Nickerson-Kveim or Kveim-Siltzbach test is a skin test used to detect sarcoidosis,
where part of a spleen from a patient with known sarcoidosis is injected into the skin of a
patient suspected to have the disease. If granulomas are found (4-6 weeks later), the test is
positive.

Your answer: (0 Points)

Question: A 34-year-old man presents with floaters and decreased vision in the left eye. He denies
any extra-ocular disorder. His right eye examination is normal. The left eye visual acuity measures
20/40, he has 1-2+ vitreous cells, and 3 areas of circumferentially oriented retinal whitening in the
midretinal periphery. What is the most likely diagnosis? (1 Points)
Answer:

A. Cytomegallovirus (CMV) retinopathy (0 Points)


B. Ocular toxoplasmosis (0 Points)
C. Acute retinal necrosis (ARN) (1 Points)

Answer Feedback:
The most likely diagnosis, given this constellation of findings in an otherwise healthy patient,
is acute retinal necrosis (ARN). All of the alternative diagnoses given can mimic some
features of ARN. CMV develops in immunosuppressed hosts, such as those with AIDS or
prior organ or bone-marrow transplant. In the immunosuppressed host, anterior chamber and
vitreous cellular reaction is minimal or absent. Toxoplasmosis presents with a single region of
chorioretinitis, unless it complicates an immunosuppressed host. Lymphoma is rare in this
age group.

D. Lymphoma (0 Points)

Your answer: (0 Points)

Print

AAO Uveitis 10 Short

Total Questions: 10 (10 Points)


Correctly Answered Questions: 0 (0 Points)
Percentage Score 0%

Questions

Question: What disorder is best characterized by a unilateral, mild, anterior uveitis with a sentinel
keratic precipitate (KP) and high intraocular pressure (IOP). (1 Points)
Answer:

A. Juvenile idiopathic arthritis-associated anterior uveitis (0 Points)


B. Glaucomacyclitic crisis (Posner-Schlossman syndrome) (1 Points)
Answer Feedback:
Increased intraocular pressure in anterior uveitis can be secondary to chronic inflammatory
damage to angle structures, iris bombe, or corticosteroids, however, in the setting of mild
uveitis and increased pressure at the beginning of an episode, viral anterior uveitis must be
considered. Glaucomacyclitic crisis is a self-limited, mild, unilateral anterior uveitis with
increased intraocular pressure, not secondary to corticosteroids. Viruses have been
implicated, although due to the self-limited nature of the process and the facts that CMV may
be the virus involved, at least in some cases, and that oral ganciclovir is not as well tolerated
as acyclovir, antiviral therapy is not often used. Fuchs heterchromic idirdocycloitis is a chronic
anterior (and sometimes intermediate) uveitis. There are diffuse stellate keratic precipitates.
The uveitis with juvenile idiopathic arthritis is chronic and bilateral. Sarcoidosis is rarely if ever
associated with a mild recurrent anterior self limited uveitis with increased intra-ocular
pressure without corticosteroid use.

C. Sarcoidosis (0 Points)
D. Fuch's heterochromic iridocyclitis (0 Points)

Your answer: (0 Points)

Question: A 32-year-old man with necrotizing scleritis and a history of recent onset sinus disease
presents with necrotizing scleritis. On physical examination, there are no other systemic complaints or
findings. What laboratory test is most likely to lead to a specific diagnosis? (1 Points)
Answer:

A. Serum rheumatoid factor and antinuclear antibodies (0 Points)


B. Serum antineutrophilic cytoplasmic antibodies (ANCA) (1 Points)

Answer Feedback:
The history is most consistent with Wegeners granulomatosis, and ANCA testing is
reasonably sensitive and specific (though not absolutlely so) with this clinical picture to
strongly suggest the diagnosis. This would be an unusual presentation (without other
systemic findings) for rheumatoid arthritis or systemic lupus erythematosis; further serum
rheumatoid factor and antinuclear antibodies are very nonsepcific, and by themselves do not
make a diagnosis. A complete blood count is very nonspecific (in partiuclar as this would not
be expected to be a systemic lymphoma or leukemia). This is not a typical presentation of
sarcoidosis; in any case the angiotensin converting enzyme is neither very sensitive nor
specific for sarcoidosis. A chest x-ray may be abnormal (or normal) in either sarcoidosis or
Wegeners granulomatosis.

C. Complete blood count (0 Points)


D. Serum angiotensin converting enzyme (0 Points)

Your answer: (0 Points)

Question: A 36-year-old African-American man presents with uveitis in his left eye. Slit lamp
examination demonstrates 2+ anterior chamber cells with 1+ flare and 2 posterior synechiae in the left
eye. Numerous anterior vitreous cells also are present just behind the lens. By ophthalmoscopy, no
chorioretinal scarring is present. The right eye also has 2 posterior synechiae, but no anterior chamber
cells or flare, and there are no anterior vitreous cells or posterior segment abnormalities. How would
you classify this uveities episode? (1 Points)
Answer:

A. Anterior and intermediate uveitis (0 Points)


B. Anterior uveitis (1 Points)

Answer Feedback:
Classifying the anatomic location of uveitis depends upon identifying the principal tissue
affected. Usually this corresponds with the site of greatest inflammation. The presence of
spill-over cells in the anterior vitreous does not qualify as intermediate uveitis, and would
potentially confound formation of an appropriate differential diagnosis. Similarly, anatomic
complications such a cystoid macular edema do not change the classification. In this case
the patient has clear evidence of anterior but not intermediate or panuveitis.

C. Panuveitis (0 Points)
D. Intermediate uveitis (0 Points)

Your answer: (0 Points)

Question: High dose corticosteroids have controlled the necrotizing scleritis associated with
Wegener's granulomatosis in your patient. What would be the preferred second intervention? (1
Points)
Answer:

A. Add a systemic immunosuprressive agent such as cyclophosphamide, infliximab, or


mycophenolate. (1 Points)

Answer Feedback:
In necrotizing scleritis associated with systemic disease, topical therapy or oral nonsteroidal
inflammatory agents would be insufficient. Indomethacin and related medications are
appropriate for mild, non-necrotizing scleritis and may help the pain, but is insufficient
treatment for necrotizing scleritis. Topical agents are also insufficient therapy (and some
would consider relatively contra-indicated, especially if any corneal involvement).They also
will not treat the associated systemic disease. Long-term high-dose corticosteroids are not
recommended as sole therapy because of significant side effects. Cytotoxic agents have long
been shown to be life saving in Wegeners granulomatosis, although now milder forms are
recognized and other agents such as mycophenolate, infliximab and rituximab may be
effective.

B. Add an oral nonsteroidal anti-inflamatory agent such as indomethacin. (0 Points)


C. Continue high-dose corticosteroids (20 mg/day) for one year and then add other agents as
needed. (0 Points)
D. Add intensive topical corticosteroid and nonsteroidal anti-inflammatory agents. (0 Points)

Your answer: (0 Points)

Question: Which of the following is a known side effect of cyclosporine therapy? (1 Points)
Answer:

A. Cystoid macular edema (0 Points)


B. Cataract (0 Points)
C. Renal insufficiency (1 Points)

Answer Feedback:
Renal insufficiency and systemic hypertension are probably the most common side effects of
concern with cyclosporine. Cataract and glaucoma and central serous choroidopathy can
occur from corticosteroid use by any route; cystoid macular edema is not a complication of
corticosteroids but is not known to be a side effect of cyclosporin.

D. Glaucoma (0 Points)

Your answer: (0 Points)

Question: What form of uveitis is most likely to follow gastroenteritis from Salmonella contaminated
food or a Chlamydial urethritis? (1 Points)
Answer:

A. Serpiginous retinochoroiditis (0 Points)


B. Acute anterior uveitis (1 Points)

Answer Feedback:
HLA-B27-associated acute anterior uveitis has followed bacterial infections, particularly gram
negative rods (and chlamydia) of mucosal surfaces. It is not an active infection, but most
likely an immune response stimulated by the mucosal infection. Although this is fairly well
established, in most patients with HLA-B27 uveitis, no such previous infection is known to
have occurred. In fact, while the association is well established, most patients with acute
anterior uveitis have no history of infection. In rare cases a gastrointestinal bacterial infection
could lead to perforation or a para-colinic abcess; in such cases metastatic endophthalmitis
must be considered. Similarly if there is severe or recurrent gastrointestinal disease,
Whipple's disease or inflammatory bowel disease must be considered. Although many forms
of uveitis may be precipitated by infection or in fact be infectious, in most cases, a specific
association has not been proven. Diffuse unilateral subacute neuroretinitis is due to a
nematode. Serpiginous and birdshot have been thought to be precipitated by infections,
possibly viral, but this has not been established, and treatment with immunosuppressive
agents appears to be effective, making active infection unlikely.

C. Diffuse unilateral subacute neuroretinitis (0 Points)


D. Birdshot retinochoroiditis (0 Points)

Your answer: (0 Points)

Question: A 28-year-old Caucasian woman with intermediate uveitis complains of paresthesisas after
a hot shower. She is from the Midwest. What disorder should be investigated as a likely cause? (1
Points)
Answer:

A. Demyelinating disease (1 Points)

Answer Feedback:
There is an association between intermediate uveitis and multiple sclerosis; it is not rare. One
can have uveitis and neuropathies with Behcets disease, but this would be very rare, and
longstanding intermediate uveitis would be a very atypical form of uveitis in Behcets disease.
Whipple's disease might be considered as there are many manifestations, which could
include intermediate uveitis and neurologic involvement, but it is very rare and much less
likely given the description of Uhtoff's sign in this patient. Intermediate uveitis, except
secondary to severe anterior uveitis, is not usually an HLA-B27-associated form of uveitis.

B. Behcets disease (0 Points)


C. Whipple's disease (0 Points)
D. HLA-B27-associated uveits (0 Points)

Your answer: (0 Points)

Question: A 29-year-old caucasian woman complains of new floaters in both eyes. She has been
camping in the eastern United States and does not remember any tick bites, but does recall a red rash
on one leg followed by bumps on her shins. She denies any sensory or motor neurologic symptoms. In
both eyes she has vitritis without any retinal or choroidal lesions. Her chest x-ray and serum
angiotensin converting enzyme (ACE) are normal. What is the preferred working diagnosis? (1 Points)
Answer:

A. Sarcoidosis (0 Points)
B. Multiple sclerosis-associated uveitis (0 Points)
C. Systemic lupus erythematosus (0 Points)
D. Lyme disease (1 Points)

Answer Feedback:
Given that the patient went camping in a Lyme endemic area and had a rash and symptoms
of erythema nodosum, the diagnosis of Lyme disease is highly likely. However, ACE and
chest x-ray are fair screening tests for sarcoidosis, which lowers this the likelihood of this
disorder. However, if Lyme titers are negative, additional testing for sarcoidosis should be
pursued. Demyelinating disease is unlikely. With this constallation of signs and symptoms,
Lyme disease is more likely. Systemic lupus erythematosis is unlikely to present in this
manner.

Your answer: (0 Points)

Question: A 32-year-old Hispanic woman with prior multifocal choroiditis and panuveitis presents with
a sudden onset of vision loss OD. She has been off any therapy for 4 years. On examination, visual
acuity is 20/200 OD with no anterior chamber inflammation or vitritis. What findings would you expect
on a fluorescein angiogram? (1 Points)
Answer:

A. Early hyperfluorescence with late leakage in the macula (1 Points)

Answer Feedback:
The most common reason for sudden vision loss in multifocal choroiditis with panuveitis is a
choroidal neovascular membrane. Such subretinal neovascular membranes also can be seen
in punctate inner choroidopathy and less commonly in birdshot chorioretinopathy. If there
were active inflammation, more gradual vision loss, perhaps even to 20/200, could be from
macular edema, but that would cause a different pattern of petaloid leakage gradually
increasing on angiography. Ocular coherence tomography may also be helpful. Late leakage
of the optic nerve head would not be expected as the optic nerve is not typically involved,
although with sufficient active inflammation a papillitis is possible. Early hyperfluorescence
with late decreasing hyperfluorescence may be seen if there is a window defect from retinal
pigment epithelial changes that can be seen in this disease but this would not explain sudden
vision loss. Early blockage and late staining is more characteristic of other entities such as
acute posterior multifocal placoid pigment epitheliopathy.

B. Early hyperfluorescence in the macula with late decreasing hyperfluorescence (0 Points)


C. Early blockage and late staining in the macula (0 Points)
D. Late leakage of the optic nerve (0 Points)

Your answer: (0 Points)

Question: In multifocal choroiditis and panuveitis, which clinical finding would most strongly influence
you to recommendation chronic systemic immunosuppression? (1 Points)
Answer:

A. Anterior chamber inflammation (0 Points)


B. Cystoid macular edema (0 Points)
C. Progressive subretinal fibrosis (1 Points)

Answer Feedback:
Subretinal fibrosis is a very difficult and potentially blinding complication of posterior uveitis. If
there is ongoing inflammation in the setting of progressive fibrosis, then aggressive therapy
must be considered, although indeed it is unclear whether it is effective it is in all cases. It is
important to determine whether choroidal neovascular membrane is present, which may need
more urgent therapy than the slow onset of most immunosuppressive therapy (including
possibly anti-VEGF therapies). It is not established whether intravitreal corticosteroids, such
as a flucinolone implant, may be an alternative once control is obtained.Both anterior uveitis
and cystoid macular edema may require chronic immunosuppressive therapy, or may be
controlled by other intervention, but are not the strongest indications for such therapy in this
disease.

D. Choroidal neovascularization (0 Points)

Your answer: (0 Points)

Print

You might also like